which one is true and why

Which One Is True And Why

Answers

Answer 1

Answer:

the first one is true

Step-by-step explanation:

√36>5 = 6>5

7<√10 = 7<3.162

√9>√25 = 3>5

Therefore, the first one is correct


Related Questions

Juan wants to buy a video game for $63. He saves $12 every Friday. Part A. Create an equation to represent Juan's total savings, y, in dollars, after x Fridays. y​

Answers

Answer:

Answer in explanation.

Step-by-step explanation:

An equation to represent Juan's total savings y, in dollars, after x Fridays will be;

⇒ y = 12x

What is an expression?

Mathematical expression is defined as the collection of the numbers variables and functions by using operations like addition, subtraction, multiplication, and division.

Given that;

Juan wants to buy a video game for $63.

And, He saves $12 every Friday.

Now,

Let total dollars after x Friday = y

And, He saves $12 every Friday.

So, We can formulate;

⇒ Total money (y) = 12x

⇒ y = 12x

Thus, An equation to represent Juan's total savings y, in dollars, after x Fridays will be;

⇒ y = 12x

Learn more about the mathematical expression visit:

brainly.com/question/1859113

#SPJ2

Evaluate this function: h(t) = |t+2| + 3; Find h(6)

Answers

Answer:

Step-by-step explanation:

h(6) = |6 + 2| + 3 = |8| + 3 = 8 + 3 = 11

e/kaid_274404147475337871026587/assig!
Khan Academy
Percent word problems
Of the 50 U.S. states, 4 have names that start with the letter W.
What percentage of U.S. states have names that start with the letter W?
0
Stuck? Review related articles/videos or use a hint.

Answers

Answer:

Step-by-step explanation:

4/50 = 0.08

0.08(100) = 8%

Solve the equation: 3.017 + k = 5.134

Answers

Answer:

k=2.117

Step-by-step explanation:

Answer:

3.017 + k = 5.134

So, k = 5.134 - 3.017

So, k = 2.117

What’s the slope of 10,6 and 4, 1.2

Answers

let's firstly change the 1.2 to a fraction

[tex]1.\underline{2}\implies \cfrac{12}{1\underline{0}}\implies \cfrac{6}{5} \\\\[-0.35em] ~\dotfill\\\\ (\stackrel{x_1}{10}~,~\stackrel{y_1}{6})\qquad (\stackrel{x_2}{4}~,~\stackrel{y_2}{\frac{6}{5}}) \\\\\\ \stackrel{slope}{m}\implies \cfrac{\stackrel{rise} {\stackrel{y_2}{\frac{6}{5}}-\stackrel{y1}{6}}}{\underset{run} {\underset{x_2}{4}-\underset{x_1}{10}}}\implies \cfrac{~~ \frac{6-30}{5}~~}{-6}\implies \cfrac{~~ \frac{-24}{5}~~}{-6}\implies \cfrac{~~ -\frac{24}{5}~~}{-\frac{6}{1}}[/tex]

[tex]-\cfrac{\stackrel{4}{~~\begin{matrix} 24 \\[-0.7em]\cline{1-1}\\[-5pt]\end{matrix}~~}}{5}\cdot -\cfrac{1}{\underset{1}{~~\begin{matrix} 6 \\[-0.7em]\cline{1-1}\\[-5pt]\end{matrix}~~}}\implies \boxed{\cfrac{4}{5}}[/tex]

Jason gets 15 gallons of gas for $3.90 a gallon which gives him 20 miles to the gallon. How many miles can Jason travel on one tank of Jason?

Answers

0.75 miles
15 divided by 20 as its in proportion

Answer:

0.75 mi.

Step-by-step explanation:

15/20=3.90/x

What is 75.65 rounded to the nearest whole number?​

Answers

It is 76 as the other person said, what I like to think to help me remember is five or more raise the score, 4 or less, let it rest

geomtry plzzz help 15 points

Answers

Answer:

19

Step-by-step explanation:

they give you xz, and they want half, so just divide 38 by 2

How many pounds are in 1 1/2 pounds and 8 ounces? There are _____ pounds in 1 1/2 pounds and 8 ounces.

Answers

Answer:

Hope this helps

Step-by-step explanation:

Since 16 ounces equal 1 pound, we’ll add 16 + 16, which is 32 ounces. For one half of a pound, simply divide the amount of ounces in a pound (16) by half. 16 divided by 2 is 8, so in total your answer would be 40 ounces in 2 1/2 pounds.

The width of a triangle is six more than twice the height. The area of the triangle is 88in2. Find the height and width of the triangle.

Answers

Answer:

w = 22 in, h = 8 in.

Step-by-step explanation:

Reading the problem, I immediately see two equations. This hints me towards a system of equations problem.

Statement 1 is The width of a triangle is six more than twice the height.

The width (w) of a triangle is (=) six (6) more (+) than twice (2) the height (h).

Equation 1: w = 6 + 2h

Statement 2 is The area of the triangle is 88 in2.

The formula for the area of a triangle is [tex]\frac{wh}{2}[/tex]. So, we get our second equation.

Equation 2: wh/2 = 88

We see that we have the same number of equations as unknowns, two. This is important as it lets us know that we can find two unknowns in the two equations. If there are more unknowns than equations, then we cannot find the values of all the variables. Look for another equation in the problem. However, if the # equations greater than or equal to number of unknowns, then we are ready to start solving!  So, we can now move to solving.

Equation 1 isolates one of the unknowns (w). So, we can replace w in the second equation with the right side of eq 1. Then, we simplify for h.

[tex]\frac{wh}{2} = 88[/tex]

[tex]\frac{(6 + 2h)h}{2} = 88[/tex]

[tex]\frac{6h + 2h^2}{2} = 88[/tex]

[tex]6h + 2h^2 = 176[/tex]

[tex]2h^2 + 6h - 176 = 0[/tex]

[tex]h = -11, 8[/tex] (assuming you know how to solve quadratics)

Since we know that heights of triangles cannot be negative, we find that [tex]h = 8[/tex] in.

For the final step, we plug [tex]h[/tex] into our first equation to find [tex]w[/tex].

[tex]w = 6 + 2h[/tex]

[tex]w = 6 + 2(8)[/tex]

[tex]w = 6 + 16[/tex]

[tex]w = 22[/tex]

So, the answer is w = 22 in, h = 8 in.

To check if we were right, we can plug the values into both equations to see if they are equal. I will skip equation 1 since we used that equation "as-is" to find w.

[tex]\frac{wh}{2} = 88[/tex]

[tex]\frac{22 * 8}{2} = 88[/tex]

[tex]11 * 8 = 88[/tex]

[tex]88 = 88[/tex]

The solution checks out!

2x+5y=(-24),3x+y=(-10) solve for using elimination method

Answers

Answer:

x equals -2 and y equals−4

31 divided by 943.89

Answers

Answer:

0.032

Step-by-step explanation:

Answer:  0.03284281007

Step-by-step explanation:


A high school track team's long jump record is 22 ft 6 3/4in. This year, Arthur's best long jump is 21 ft 10 1/2in. If long jumps are measured to the nearest quarter inch, how much farther must
Arthur jump to break the record?
To beat the school long jump record,

Arthur must jump an additional ___inches.

(Type a whole number, fraction, or mixed number.)

Answers

Arthur must jump an additional 8.25 inches so as to break the record.

1 foot =  12 inches

Long jump record = 22 ft 6 3/4in = 22 ft 6.75 in = (22ft * 12 in per ft) + 6.75 in

Long jump record = 270.75 in

Arthur's long jump record = 21 ft 10 1/2in = 21 ft 10.5 in = (21ft * 12 in per ft) + 10.5 in

Arthur's long jump record = 262.5 ft.

To break the record, the distance needed to be jump by Arthur = 270.75 in - 262.5 ft. = 8.25 in

Arthur must jump an additional 8.25 inches so as to break the record.

Find out more at: https://brainly.com/question/20796404

if a chocolate cake is divided into six equal slices and each slice has 280 calories

Answers

Answer:

Whats the question?

Step-by-step explanation:

Answer:

1,680

Step-by-step explanation

6 slices each 280 calories, 280 x 6 = 1,680

geomtry plz help 15 points

Answers

Answer:

it should be 60°. the single marks on each side means they are all equal

Answer:

m∠L=60º

Step-by-step explanation:

The triangle is an equilateral triangle meaning that all angles and sides are equal.

m∠L=60º

Hope this helps :)

Note: Enter your answer and show all the steps that you use to solve this problem in the space provided.

You buy clothing at a sale. You buy a sweater at
7
10
of its original price of $35. Answer each question and show all your work.

a. How much money did you spend?
b. How much money did you save?
c. What fraction of the total original price did you save?

Answers

The amount that will be spent is $24.50, the amount saved will be $10.50, and the percentage saved will be 30%.

The amount that will be spent will be:

= 7/10 × $35 = $24.50

The amount that will be saved will be:

= $35 - $24.50 = $10.50

The fraction of the original price that is saved will be:

= 10.50/35 × 100

= 30%

Read related link on:

https://brainly.com/question/25763817

which number will reach if we move 6 steps to the right -3

Answers

Answer:

If you meant -3 + 6, then the correct answer is 3.

You should put some extra effort in composing questions with all the required information, like a picture maybe.

Answer:

your answer has to be 3

Step-by-step explanation:

if you were to move to the right in number line you have to add so -3+6 is equals to 3

what is 4% of 32? is it a percent, base, or amount

Answers

Answer:

1.28 - amount

Step-by-step explanation:

You would multiply 32 times 0.04 and get 1.28 which I assume would be an amount.

The amount of 4% of 32 is 1.28.

We have to determine what is 4% of 32.

What is the percentage?

This free percentage calculator computes a number of values involving percentages, including the percentage difference between two given values.

Suppose it is an x percent.

Therefore x multiply 32 times 0.04 and get 1.28 an amount.

To learn more about the percentage of visits:

https://brainly.com/question/24304697

#SPJ2

Suppose a jar contains 17 red marbles and 32 blue marbles. If you reach in the jar and pull out 2 marbles at random, find the probability that both are red.

Answers

There are 49 total marbles, the chance of you getting a red is 17/49, and if you don’t replace the marble and pull out another one, the probability of that being red is 17/49*16/48=17/147. That’s approximately a .12% chance.
16/48=1/3

The probability that both are red marbles = [tex]\bold{\frac{17}{147} }[/tex]

What is probability?

"It is finding out the possibilities of the occurrence of an event."

Formula to find the probability of an event:

"P(A) = n(A) / n(S)

where, n(A) is the number of favorable outcomes of an event A

n(S) is the total number of outcomes for an experiment"

For given example,

A jar contains 17 red marbles and 32 blue marbles.

If we pull out 2 marbles at random, we need to find the probability that both are red.

The number of possible outcomes,

[tex]\Rightarrow n(S)=^{49}C_2\\\\ \Rightarrow n(S)=\frac{49!}{2!(49-2)!}\\\\ \Rightarrow n(S)=1176[/tex]

The number of possible outcomes of selecting both the red marbles.

[tex]\Rightarrow n(A)=^{17}C_2\\\\ \Rightarrow n(A)=\frac{17!}{2!(17-2)!}\\\\ \Rightarrow n(A)=136[/tex]

The probability that both the marbles are red,

[tex]\Rightarrow P(A)=\frac{n(A)}{n(S)}\\\\ \Rightarrow P(A)=\frac{136}{1176}\\\\ \Rightarrow P(A)=\frac{17}{147}[/tex]

Therefore, the probability that both are red marbles = [tex]\bold{\frac{17}{147} }[/tex]

Learn more about the probability here:

brainly.com/question/11234923

#SPJ2

in a supermarket, credit card purchases have an additional charge of 8% of the purchase made. If I bought 2 suitcases that cost s / .165.60 each and paid with a credit card, how much do I have to pay?

Answers

9514 1404 393

Answer:

  357.70

Step-by-step explanation:

The final cost will be ...

  (number of suitcases) × (cost per suitcase) × (1 +8%)

  = 2(165.60)(1.08) = 357.70

Write the equation of the function graphed below:

a. f(x) = 2x−−√3 –4
b. f(x) = 2x−−√3 + 4
c. f(x) =x−−√3 + 4
d. f(x) =x−−√3 – 4

Answers

Answer:

I have a couple questions before I answer, what do the 2 dashes represent and are the 4s also under the square root?

Alf needs to borrow $15,000 to pay for his college tuition. He can borrow the money from his
parents at a rate of 3.55% interest compounded annually for 4 years, or he can borrow from his
local bank at a rate of 3.50% interest compounded continuously for 4 years
Manipulation of Numerical Data

Answers

Alf would prefer to borrow from his parents because their rates are more favourable.

The future value of the loan if he borrows from his parents is given by this formula: A(1 + r)^n

Where:

A = amount  

R = interest rate  

N = number of years  

$15,000(1 + 0.0355)^4 = $17,246.13

The future value of the loan if he borrows from the local bank is given by this formula: : A x e^r x N

Where:

A= amount e = 2.7182818 N = number of years r = interest rate

$15,000 x  2.7182818^0.035 x 4 = $62,137.18

To learn more about future value, please check: https://brainly.com/question/14640433

Match the event with the most appropriate probability word: The roll of a normal fair dice will show a 7.

A. Certainly
B. Likely
C. Evens
D. Unlikely
E. Impossible
F. Other

Answer this, plz.

Answers

Answer:

Option 3

Evens

. . . . . .. . ..

35. In her rough work, Janu added 2389 and 740 as shown here.
Is it okay to write the number 740 this way in the addition?
(Hint: You do NOT need to calculate)
(A) Yes, because 0 has no value so 74 is the same as 740.
(B) No, because the answer will be wrong if 0 is not written.
(C) No, because the digits 7 and 4 have NOT been arranged according to their value.
(D) Yes, because 7 is added with the hundred's digit and 4 with the ten's digit.

Answers

Answer:

D

Step-by-step explanation:

Which graph represents 6x-2y>-11

Answers

Answer:

Answer:

C

Step-by-step explanation:

Remember when using the inequality sign < or > the line will be dotted so we can immediately eliminate answer choices b and d

Now let us solve for y

6x-2y>-11

step 1 subtract each side by 6x

now we have

-2y>-11-6x

step 2 divide each side by -2

-11-/2=11/2

-6/-2=3

IMPORTANT -when you divide inequalities by a negative number you flip the inequality sign

So now we have

See image below for reference

When the inequality is facing x the solutions are going to be on top of the line

hope this helps!

Step-by-step explanation:

geomtry plz help 15 points

Answers

Answer:

m∠O = 41°

Step-by-step explanation:

∠NOM=∠NMO=(4y-15)° (base angles of isos triangle)

7y+2(4y-15)=180 (angle sum of triangle)

  7y+8y-30=180

       15y-30=180

             15y=180+30

                  =210

                y=210÷15

                  =14

Hence, m∠O = (4y-15)°

                      = [4(14)-15]°

                      = (56-15)°

                      = 41°

2x - 5y = 6 and -2x + 7y = 14

*Using elimination*​

Answers

2x-5y+ (-2x) + 7y= 6+14
2y = 20
y= 10
2x-5(10)=6
2x= 56
x= 28
here is the ans.hope this helps!

What is the coefficient of x in expression of 5mx​

Answers

Coefficient of this expression is 5 only.

- BRAINLIEST answerer

g A long-term study has revealed that a test for cancer in men is very effective. The study shows that 89% of the men for which the test is positive actually have cancer. If a man selected at random tests positive for cancer with this test, what is the probability that he does not have cancer

Answers

The answer is 89% because I had that Same question and it was 89%

Last year at a certain high school, there were 100 boys on the honor roll and 60 girls on the honor roll. This year, the number of boys on the honor roll decreased by 11% and the number of girls on the honor roll decreased by 10%. By what percentage did the total number of students on the honor roll increase? Round your answer to the nearest tenth (if necessary).​

Answers

Answer:

-11 boys -6 girls

Step-by-step explanation:

Other Questions
Why does acid rain occur in Earth? plzzz help. What is the acceleration as an object goes from +20 m/s to +26 m/s over 1.4 s? An art club sells 42 large candles and 56 small candles.a. Use the Distributive Property to write and simplify an expression for the profit. expression for the total profit: _(_-x) + (_-y) simplified expression for the total profit: b. A large candle costs $5, and a small candle costs $3. What is the clubs profit? club's profit: $ What is the best reason why people started to settle in one place and population started to increase? Write an inheritance hierarchy for classes Quadrilateral, Trapezoid, Parallelogram, Rectangle and Square. Use Quadrilateral as the superclass of the hierarchy. Create and use a Point class to represent the points in each shape. Make the hierarchy as deep (i.e., as many levels) as possible. Specify the instance variables and methods for each class. The private instance variables of Quadrilateral should be the x-y coordinate pairs for the four endpoints of the Quadrilateral. Write a program that instantiates objects of your classes and outputs each objects area (except Quadrilateral how are the midwest and the northeast regions similar? can you write the answer for lot of points given the sequence tn defined recursively below, find t4. How are Jules and Ilana related? What do the Mississippi and Amazon river have in common? Which of the following are required for plants to carry out photosynthesis? How is literature impacted in the united states. Please take a look at the picture 1. Abraham said, My SON, God will provide HIMSELF a lamb.2. In the BEGINNING was the WORD, and the Word was with GOD, and the Word was God.3. To QUIT is not an OPTION.4. The terrible HOUR is still only sixty MINUTES.5. There is sweet JOY which comes to us through SORROW.Identify the bold nouns as:A. SubjectB. VerbC. Direct ObjectD. AppositiveE. Predicate NominativeF. Direct AddressG. Indirect ObjectH. Object of PrepositionI. Adjective (verbal used as an adjective) Circle the multiplication problem that does not belong with the other three. explain 128=,224=,422=,333= Which shows the prodcuts orderd from least to greatest Round off the decimal 55.345 to 2 decimal places. if a number became 2.5 times smaller, what precent was it decreased by If you have to decide to claim a credit or deduction on your taxes which should you take? Write a program that lists all ways people can line up for a photo (all permutations of a list of strings). The program will read a list of one word names (until -1), and use a recursive method to create and output all possible orderings of those names separated by a comma, one ordering per line.